Difference between revisions of "1967 AHSME Problems/Problem 26"

(Created page with "== Problem == If one uses only the tabular information <math>10^3=1000</math>, <math>10^4=10,000</math>, <math>2^{10}=1024</math>, <math>2^{11}=2048</math>, <math>2^{12}=4096</ma...")
 
(Solution)
Line 9: Line 9:
  
 
== Solution ==
 
== Solution ==
<math>\fbox{C}</math>
+
Since 1024 is greater than 1000.
 +
 
 +
log 1024 > 3   
 +
 
 +
10 * log 2 > 3
 +
 
 +
and log 2 > 3/10.
 +
 
 +
 
 +
Similarly, 8192 < 10000, so log 8192 < 4
 +
 
 +
13 * log 2 < 4
 +
 
 +
and log 2 < 4/13
 +
 
 +
 
 +
Therefore  3/10 < log 2 < 4/13
 +
so the answer is <math>\fbox{C}</math>
  
 
== See also ==
 
== See also ==

Revision as of 21:56, 9 July 2015

Problem

If one uses only the tabular information $10^3=1000$, $10^4=10,000$, $2^{10}=1024$, $2^{11}=2048$, $2^{12}=4096$, $2^{13}=8192$, then the strongest statement one can make for $\log_{10}{2}$ is that it lies between:

$\textbf{(A)}\ \frac{3}{10} \; \text{and} \; \frac{4}{11}\qquad \textbf{(B)}\ \frac{3}{10} \; \text{and} \; \frac{4}{12}\qquad \textbf{(C)}\ \frac{3}{10} \; \text{and} \; \frac{4}{13}\qquad \textbf{(D)}\ \frac{3}{10} \; \text{and} \; \frac{40}{132}\qquad \textbf{(E)}\ \frac{3}{11} \; \text{and} \; \frac{40}{132}$

Solution

Since 1024 is greater than 1000.

log 1024 > 3

10 * log 2 > 3

and log 2 > 3/10.


Similarly, 8192 < 10000, so log 8192 < 4

13 * log 2 < 4

and log 2 < 4/13


Therefore 3/10 < log 2 < 4/13 so the answer is $\fbox{C}$

See also

1967 AHSME (ProblemsAnswer KeyResources)
Preceded by
Problem 25
Followed by
Problem 27
1 2 3 4 5 6 7 8 9 10 11 12 13 14 15 16 17 18 19 20 21 22 23 24 25 26 27 28 29 30
All AHSME Problems and Solutions

The problems on this page are copyrighted by the Mathematical Association of America's American Mathematics Competitions. AMC logo.png